Step 2 CK (CCSSA) Form 8

These answer explanations are and always will be free. However, given multiple email requests, I will post my Venmo (@Adam-Zakaria-SLO) if you want to send a few dollars to show your support for the website.

I also offer reasonably priced Study Guides and Personalized Study schedules, so please reach out using the Tutoring menu option or Study Guides and Personalized Study schedules menu option listed above if you would like personalized support.

Furthermore, I offer personal statement and application review services for residency applicants, so please reach out using the “Residency Advising and Application Preparation” menu option above if interested.

Lastly, please check out my Youtube channel (https://www.youtube.com/channel/UCT1Ukl4pm5QK9iw6h4MB_Hw/playlists) and the “Biostatistics Curriculum” option above for free videos and practice questions reviewing all the essential biostatistics topics covered on NBME exams. Good luck with your exams!


Exam section 1:

1. A 57-year-old woman comes to the physician because of a…

Toxin-induced inflammation of the colonic mucosa

  • Patient with recent antibiotic usage develops severe diarrhea and abdominal pain, which is most consistent with C. difficile infection
  • Key idea: C. difficile leads to diarrhea by producing two main toxins that lead to enterocyte damage
  • Key idea: Toxic megacolon can be seen secondary to C. difficile infection and ulcerative colitis (at least for NBME purposes)

2. An asymptomatic 37-year-old woman comes to the physician…

Discontinue the oral contraceptive

  • Contraindications to combined contraception includes (1) Women > 35 yo who smoke (which is case with this patient) (2) Migraines with aura (3) Hypercoagulable risk factors (<3 weeks postpartum, prolonged immobilization, history of DVT/PE, blood disorders [factor V leiden, etc.]) (4) Active breast cancer (5) Liver disease (liver involved in metabolizing estrogen)

3. A 52-year-old woman with a 40-year history of type 1 diabetes…

Abnormal pressure distribution caused by sensory neuropathy

  • 3 main types of lower extremity ulcers include (1) Neuropathic ulcers: Seen in diabetic patient, occur on the sole of the foot especially where lots of pressure is placed while walking (such as proximal to big toe) (2) Arterial ulcers: Patient with signs of peripheral vascular disease (claudication, shiny hairless legs) who has ulcer with necrosis often at tips of the toes (3) Venous ulcers: Often seen in patient with chronic lower extremity edema and leads to ulceration on the medial aspect of the leg

4. A 67-year-old woman comes to the physician for preoperative…

Pulmonary function tests

  • Tricky, nuanced question because contraindication to patient receiving a lung resection is if their residual FEV1 will be less than 800 mL following the surgery
  • Normal FEV1 in a healthy woman is typically around 3 L, so in this patient is likely close to 750 mL since she has an FEV1 that is 25% of predicted; therefore, this patient would likely not be able to tolerate any sort of lung resection because she is already barely hanging on in terms of her ventilatory capabilities
  • Could also approach this question through process of elimination because ABG shows mild CO2 retention (seen in many COPD patients), ejection fraction cut-off for surgery often <35%, cardiac stress test was normal and patient has had DVTs for a long time

5. A 37-year-old woman, gravida 2, para 2, comes to the physician…

Adenomyosis

  • Woman in her late 30s with previous pregnancies presents with progressive menstrual pain and a soft, diffusely enlarged uterus most consistent with adenomyosis (abnormal endometrial tissue within the uterine myometrium)
  • Key idea: Adenomyosis can lead to dysmenorrhea, heavy menstrual bleeding and chronic pelvic pain
  • Key idea: Risk factors for adenomyosis includes age > 40, multiparity and/or prior uterine surgery
  • Key idea: For the NBME exams, a diffusely enlarged uterus is either pregnancy or adenomyosis, whereas an enlarged irregular uterus is consistent with fibroids

6. A 27-year-old man with schizophrenia is brought to the …

Adverse effect of haloperidol

  • Young man who recently started treatment with haloperidol presents with rigidity, neck rigidity and oculogyric crisis (eyes displaced upwards) most consistent with acute dystonia
  • Key idea: Important to contrast with neuroleptic malignant syndrome, which would lead to constellation of symptoms remembered with Malignant FEVER (Myoglobinuria, Fever, Encephalopathy, Vitals unstable, increased CK Enzymes and Rigidity)
  • Key idea: 4 main extrapyramidal effects of antipsychotic use, especially with high potency antipsychotics such as haloperidol, are (1) Acute dystonia (2) Akathisia (3) Parkinsonism (4) Tardive dyskinesia

7. An asymptomatic 27-year-old man comes to the physician 1…

Influenza virus, 23-valent pneumococcal, and hepatitis B vaccines

  • All patients (HIV or not) should receive an annual influenza vaccine
  • Patients with HIV should receive Strep Pneumo PCV13 followed by the 23-valent PPSV23 8 weeks later and again in 5 years and at age 65
  • Based on patient’s serologies (negative Hep B surface antibody), he needs to be vaccinated against hepatitis B
  • Note: Patients are not given PCP prophylaxis until CD4 < 200 and T. gondii prophylaxis until CD4 < 100

8. A 62-year-old man comes for evaluation prior to undergoing…

Parenteral cefazolin

  • Current guidelines recommend cefazolin or cefuroxime administered within 60 minutes of skin incision as prophylaxis during hip or knee arthroplasty and should be discontinued within 24 hours
  • Key idea: Antibiotic prophylaxis to prevent surgical site infections are given in certain procedures, most commonly major/long surgeries
https://www.aafp.org/afp/2011/0301/p585.html

9. An otherwise healthy 37-year-old woman comes to the physician…

Endometrial biopsy

  • Young woman with a strong family history of cancer compatible with Lynch syndrome (colon cancer, endometrial cancer, ovarian cancer) who has developed irrregular, heavy vaginal bleeding concerning for possible endometrial cancer
  • Indications for endometrial biopsy include (1) Woman over 45 years old with abnormal uterine bleeding or postmenopausal bleeding (2) Woman under 45 with abnormal uterine bleeding with unopposed estrogen (PCOS, obesity), failed medical management or Lynch syndrome

10. A 72-year-old man is transferred to the emergency department…

Arterial embolus

  • Older man with atrial fibrillation who is not on prophylactic anticoagulation who presents with non-specific lethargy, hypotension, abdominal tenderness with guarding, increased WBC count and surgical findings of dark necrotic bowel, most consistent with an arterial embolus (most likely from the left atrial appendage in setting of atrial fibrillation)
  • Adhesions would lead to small bowel obstruction (pain, distention, obstipation), and C. difficile would lead to pseudomembrane image on NBME

11. An 87-year-old man comes to the physician because of problems…

Normal aging

  • 87 year old man who is independent in ADLs (eating, bathing, toileting, transferring, getting dressed) and iADLs (shopping, using phone, driving, handling finances, etc.), A&O x 3, recalls 2/3 objects after 5 minutes and has no focal neurologic findings most consistent with normal aging or mild cognitive impairment
  • Key idea: Patients with mild cognitive impairment will have mild decline in at least 1 cognitive domain, but will have normal functioning in all activities of daily living with compensation (taking a list to the store), whereas patients with dementia will have global cognitive impairment and marked functional impairment
  • To differentiate between Alzheimer’s and vascular dementia, look for other neurologic features (pronator drift, weakness, etc.) which is more consistent with vascular dementia

12. A 37-year-old nulligravid woman comes to the physician…

Intraductal papilloma

  • Most common cause of unilateral, bloody breast discharge is intraductal papilloma
  • Key idea: Intraductal papilloma often does not have a mass on physical exam or mammography findings

13. A 47-year-old man comes to the emergency department 2…

Air embolism

  • Patient who has recently gone diving (which should be sounding off alarms about decompression sickness (AKA “The bends”)) who presents with weakness and neurologic symptoms concerning for decompression sickness
  • Key idea: Decompression sickness occurs when nitrogen dissolved in blood at high pressures of deep sea diving forms bubbles as pressure decreases due to rapid ascent to the surface and the bubbles lead to obstruction of blood vessels
  • Key idea: Potential symptoms of decompression sickness include signs similar to stroke (due to blood vessel blockage), flu-like symptoms, and swelling/pain in muscles and joints (remember association of decompression sickness with avascular necrosis of bone)

14. A 19-year-old man is brought to the emergency department…

Psychogenic polydipsia

  • Young male with excessive thirst and urination for 6 months who presents with seizures found to have hypo-osmolar hyponatremia, most consistent with psychogenic polydipsia
  • Key idea: This demographic is classically when signs/symptoms of schizophrenia develop, and patients with schizophrenia are at increased risk for psychogenic polydipsia
  • Key idea: If patient has hypo-osmolar hyponatremia and a urine osmolality < 100, then they almost certainly have psychogenic polydipsia or beer potomania
https://www.aafp.org/afp/2015/0301/p299.html

15. A 52-year-old man comes to the physician because of a 3-day…

Proliferation of hematopoietic progenitor cells

  • Middle-aged man with fever, lymphadenopathy and splenomegaly found to have elevated leukocyte count with increased eosinophils, myeloblasts, metamyelocytes and myelocytes most concerning for chronic myeloid leukemia
  • Key idea: Ways to differentiate between chronic myeloid leukemia and a leukemoid reaction include leukocyte alkaline phosphatase (LAP) levels (elevated in leukemoid reaction and decreased in CML), looking for basophilia (present in CML) and looking at the types of neutrophil precursors present (metamyelocytes > myelocytes in leukemoid reaction and metamyelocytes < myelocytes in CML)
  • Note: Metamyelocytes are more mature than myelocytes
  • Key idea: Can often tell they type of leukemia based on patient demographics with ALL seen in patients 0-14, AML seen in patients 15-60, CLL seen in patients 60+ and CML seen in patients 40-59

16. A previously healthy 23-year-old man comes to the physician…

Consistent use of condoms

  • Young patient with high-risk sexual behavior who presents with fever, malaise, maculopapular rash over the palms and soles and diffuse lymphadenopathy, which is most consistent with syphilis
  • Key idea: 3 major infectious causes of rash over the palms and soles can be remembered with CaRS (Coxsackie A, Rickettsia Rickettsii (Rocky Mountain Spotted Fever) and Syphilis)
  • Important to contrast syphilis and Rocky Mountain Spotted Fever, with syphilis more commonly leading to a maculopapular rash and diffuse lymphadenopathy (with epitrochlear lymphadenopathy basically being pathognomonic) and Rocky Mountain spotted fever commonly leading to a purpuric rash that spreads from the hands/feet to the trunk

17. A 16-year-old girl comes for a routine health maintenance…

Transesophageal echocardiography

  • 16 year old girl with primary amenorrhea (girl without secondary sexual changes by 13 or menarche by 15), short stature, and concern for coarctation of the aorta (hypertension, upper extremity pulses > lower extremity pulses, systolic murmur), most consistent with Turner syndrome
  • Coarctation of the aorta needs to be worked up with an echocardiography and then operatively repaired

18. A 32-year-old man comes to the physician because of…

Propranolol

  • Young man with history compatible with migraines (throbbing headaches associated with aura [nausea, vomiting, photophobia]) that are increasing in frequency and are refractory to abortive/acute treatments, making this patient a good candidate for prophylactic migraine medication
  • Key idea: Indications for migraine prophylactic medications include (1) Headaches > 4 times per month or long-lasting [>12 hours] (2) Disabling symptoms (missing work) (3) Refractory to abortive medications
  • Key idea: Potential prophylactic migraine medications include Topiramate, Valproate, Beta blockers (such as propranolol), calcium channel blockers or amitriptyline (TCA antidepressant)

19. A 77-year-old man is brought to the physician for a follow-up…

Begin isoniazid and vitamin B6 therapy

  • 77-year old patient with a positive PPD skin test (>10 mm positive in patient living in a high-risk facility) who is asymptomatic most consistent with latent tuberculosis
  • Key idea: 3 treatment options for latent tuberculosis are (1) Isoniazid and rifapentine weekly for 3 months (2) Isoniazid for 6-9 months (3) Rifampin for 4 months
  • Key idea: Patients with diabetes, uremia, alcoholism, malnutrition, HIV, pregnancy or epilepsy should always be given pyridoxine (vitamin B6) along with isoniazid due to increased risk of B6-deficiency while on isoniazid
  • Key idea: PPD induration (NOT erythema) required for positivity depends upon patient risk factors with >5 mm being positive in patients with significant immunosuppression (HIV, organ transplant, immunosuppressant meds), recent contact with patient with active TB or patients with CXR findings consistent with TB // >10 mm being positive for patients from Tb endemic countries, IVDU, residents of high-risk settings (prisons, nursing homes, homeless shelters, etc.), children < 4 years old and patients working in mycobacterial labs // >15 mm in all patients

20. A 22-year-old woman comes to the physician because of…

Deficiency of serum glucuronosyltransferase

  • Young otherwise healthy woman with short course of nausea and diarrhea has persistent indirect hyperbilirubinemia on exam with otherwise normal LFTs, most consistent with Gilbert’s syndrome (deficiency of serum glucuronosyltransferase such that you cannot efficiently conjugate bilirubin)
  • Key idea: Prevalence of Gilbert’s syndrome is ~5-10%

21. A 12-year-old boy with Marfan syndrome is found to have…

Mitral valve prolapse

  • Key idea: The two most common murmurs heard in setting of Marfan syndrome include aortic regurgitation (early decrescendo diastolic murmur heard best at left/right upper sternal border) and mitral valve prolapse (mid-systolic click followed by mid-to-late systolic murmur of mitral regurgitation heard best at the apex)
  • Key idea: HOCM and MVP are the two murmurs that improve with increases in preload (increased preload leads to less obstruction in HOCM and later MVP murmur because the LV will be more stretched and the chordae tendinae will be more taut)

22. A 3-year-old girl is brought to the emergency department 30…

Intramuscular epinephrine

  • Atopic child (eczema, allergic rhinitis) who presents with severe respiratory distress, hypotension, edema of the face and hands and subglottic narrowing on AP x-ray, most consistent with anaphylactic shock
  • Although the history of respiratory distress and stridor in a patient without childhood vaccinations should raise concern for epiglottitis (in which case we would treat with intubation + antibiotics), this diagnosis would lead to sore throat, high fever and drooling and would not as commonly lead to hypotension and would not lead to edema of the face/hands or subglottic narrowing

23. Twelve days after undergoing an orthotopic liver transplant…

Allograft rejection

  • Patient with recent liver transplant presents weeks later with fever, eosinophilia (in a patient taking prednisone where we would otherwise expect low eosinophils), elevated LFTs and a biopsy showing a mononuclear infiltrate with eosinophils in the hepatic triads, consistent with allograft rejection
  • Key idea: Graft eosinophilia is a sensitive and specific marker of acute rejection in liver allografts (https://www.ncbi.nlm.nih.gov/pubmed/9724472)

24. A 9-year-old girl is brought to the physician by her parents…

Sertraline

  • Presentation consistent with Obsessive-compulsive disorder, with the patient having distressing obsessions (frequent thoughts of dog and parents being hurt) who copes with repeated compulsions (taps foot, turns lights on and off)
  • Key idea: First line treatment in OCD is SSRIs + CBT (exposure and response prevention); second-line treatments include SNRIs and clomipramine

25. A previously healthy 52-year-old man is brought to the…

Second-degree atrioventricular block

  • Rhythm strip shows progressively prolonged PR interval until a QRS complex is dropped
  • First-degree AV block = Prolonged PR interval (>200 ms or one big box) but every p wave is followed by a QRS
  • Second-degree type 1 block = PR interval becomes increasingly prolonged until a QRS complex is dropped (p wave not followed by QRS)
  • Second-degree type 2 block = PR interval prolonged with random QRS complexes that are dropped
  • Third-degree block = Atria and ventricles beat independently of one another, with QRS complexes occurring at rates ~40 beats/min and p waves occurring at rates ~80 beats/min

26. A 7-year boy is brought to the physician because of pain and…

Topical antibiotic therapy

  • Child with pain/itching of the year with pain upon manipulation of the tinna, most consistent with otitis externa
  • Otitis externa is most commonly treated with a topical antibiotic (ciprofloxacin drops)

27. A 52-year-old man is brought to the emergency department…

Thoracoabdominal aortic dissection

  • Middle-aged man with tearing back pain unresponsive to nitroglycerin but responsive to morphine and labetalol found to have severe upper extremity blood pressure with diminished lower extremity pulses, most consistent with an aortic dissection
  • Key idea: Aortic dissection can spread to involve the coronary arteries, leading to an MI (i.e. just because a patient has chest pain and ECG changes consistent with an MI does NOT exclude aortic dissection as a diagnosis)
  • Key idea: Beta-blockers are particularly useful in setting of aortic dissection because the force of blood being ejected from the LV and hitting the aorta can further propagate the tear, with beta blockers leading to reduced contractility and reduced force of blood ejection against the aorta

28. A previously healthy 72-year-old man comes to the physician…

Pneumoconiosis

  • Elderly man with significant smoking history and occupational history of being a coal miner who presents with clubbing and hypoxia with bilateral upper lobe masses and areas of consolidation on chest x-ray most consistent with COPD and/or coal workers’ pneumoconiosis
  • Key idea: Of the common pneumoconioses, the majority affect the upper lobes (silicosis, coal workers’ lung, berylliosis) except for asbestosis (lower-lobe predominant)
  • Tuberculosis can also lead to upper lobe opacities, but it would more commonly lead to acute fever, night sweats and weight loss in a patient with exposure

29. A comatose 29-year-old woman is brought to the emergency…

Pneumothorax

  • A patient who likely came to ED with altered mental status in the setting of benzodiazepine overdose is intubated and then becomes agitated, hypoxic, tachycardic and hypotensive most concerning for a tension pneumothorax
  • Key idea: Trauma and mechanical ventilation are two major risk factors for tension pneumothorax
  • Key idea: Ventilator would have high-pressure alarms in this setting because the machine is likely trying to push the same amount of air into the lungs even though the volume has been cut in half

30. A 37-year-old woman comes to the physician because of a…

Use of a wrist splint

  • History of pins and needles in the hand that is especially bothersome at night and improve with shaking the hand are most consistent with carpal tunnel syndrome (compression of median nerve at the wrist)
  • Initially treated with wrist splinting, with patients with refractory patients receiving steroid injections or even surgery

31. The physician reads a news report about a 50% reduction…

Relative risk reduction

  • Relative risk reduction = 1 – relative risk = 1 – (.1/.2) = 1 – .5 = 0.5 = 50%

32. A 25-year-old man is brought to the emergency department…

Aspirin

  • Young healthy man who presents with chest pain and ST-elevation in an anterior distribution after using crack cocaine, most consistent with cocaine-induced MI
  • Key idea: Cocaine induced MI treated the same way as atherosclerotic MI, except that beta blockers are not used (due to theoretical risk of unopposed alpha-agonism) and benzodiazepines are used
  • Key idea: Cocaine use is associated with myocardial infarction (due to spasm of the coronary arteries) and aortic dissection (due to increased blood pressure), but this presentation more consistent with pure MI because patient has normal BP in both arms (although this is commonly seen in aortic dissections in real life, test writers often will say blood pressures are unequal) and no tearing chest pain into the back

33. A 27-year-old man comes to the physician because of a 2-month…

AIDS

  • 27 year old patient with hepatitis B (which shares many risk factors with HIV infection) presents with chronic weight loss and acute shortness of breath with cervical lymphadenopathy, white plaques in the mouth (most consistent with thrush), relatively low lymphocyte count and bilateral diffuse infiltrates on CXR most consistent with Pneumocystis pneumonia in setting of HIV/AIDs
  • Key idea: Patient with HIV is considered to have progressed to AIDS when their CD4 count drops below 200 or if they develop an AIDS-defining infection (such as PCP pneumonia)

34. A 72-year-old woman with chronic obstructive pulmonary…

Constriction of vessels in the lungs

  • Elderly woman with chronic COPD history presents with weight loss, progressive shortness of breath and light-headedness found to have JVD with RV heave and loud S2 (all components of pulmonary hypertension) and signs of congestive hepatopathy, all consistent with Group 3 pulmonary hypertension
  • Key idea: In most capillary beds of the body, decreased oxygen leads to vasodilation in order to facilitate more bloodflow to the area to maintain perfusion, but the lungs are the one exception in which decreased oxygen leads to constriction of blood vessels feeding that area so that blood is preferentially sent to well-oxygenated portions of the lung to minimize V/Q mismatch, but can lead to cor pulmonale in patients with COPD because their entire lungs are poorly oxygenated, leading to diffuse vasoconstriction and increased afterload on right ventricle

35. A 72-year-old woman is undergoing inpatient rehabilitation…

Flaccid neurogenic bladder

  • Elderly patient with recent stroke leading to residual right hemiparesis presents with frequent urinary dribbling and filled bladder (palpable smooth mass in suprapubic area), most consistent with neurogenic bladder complicated by overflow incontinence
  • Note: Detrusor instability would lead to urgency incontinence and increased vesicourethral angle associated with stress incontinence

36. A 9-year-old boy is brought to the physician by his mother…

Tourette disorder

  • In order for patient to be diagnosed with Tourette syndrome, they need to have both multiple motor tics (blinking, shoulder shrugging, sniffing, facial grimacing) and at least 1 vocal tic (grunting, snorting, throat clearing, yelling, obscenities) for at least 1 year

37. A 72-year-old man with lung cancer comes to the physician…

Repetitive nerve stimulation

  • 72 year old patient with lung cancer presents with non-fatigable weakness affecting the eyes and proximal muscles (hip extensors, eyes), autonomic symptoms (dry mouth, constipation) and loss of deep tendon reflexes, most consistent with Lambert-Eaton syndrome (which can be diagnosed with repetitive nerve stimulation)
  • Myasthenia gravis: Associated with thymoma, oculobulbar weakness that worsens with repeated use
  • Lambert-eaton syndrome: Associated with small cell lung cancer, proximal muscle weakness that improves with repetitive use AND autonomic dysfunction and decreased/absent deep tendon reflexes

38. A 42-year-old man comes to the physician because of shortness…

  • Patient with small cell lung cancer (uniform, small round cells with dark nuclei), which is often initially treated with chemotherapy followed by radiation and/or surgery
  • Patient is young and has no signs of metastasis so palliative care isn’t appropriate, hormone therapy most used for breast cancer and prostate cancer, and surgical resection is NOT used in small cell lung cancer specifically (“the cells are too small for the surgeon to see!”)

39. A previously healthy 32-year-old woman comes to the physician…

Irritable bowel syndome

  • Young healthy woman presenting with abdominal pain relieved by stools, with stool consistency being abnormal, all features of irritable bowel syndrome
  • Key idea: IBS is a diagnosis of exclusion that can lead to constipation and/or diarrhea and is often associated with (1) pain related to defecation (2) change in stool consistency (3) change in stool frequency

40. A 42-year-old woman, gravida 4, para 4, comes to the physician…

Detrusor hyperactivity

  • Patient with risk factors for spastic bladder (MS) has presentation consistent with urgency incontinence (urge to void immediately with loss of urine before reaching the bathroom at times), which is caused by detrusor hyperactivity/instability
  • Stress incontinence: Loss of urine with cough or increased abdominal pressure, caused by urethral hypermobility or sphincter deficiency
  • Overflow incontinence: Incomplete emptying of bladder leading to leak with overfilling; patient would have increased postvoid residual

41. A 42-year-old woman comes to the physician for a routine…

Nonalcoholic steatohepatitis

  • Young obese woman with no alcohol history found to have dyslipidemia and mildly elevated LFTs most consistent with non-alcoholic steatohepatitis
  • Patients use of acetaminophen not sufficient to cause liver damage (more commonly leads to fulminant liver failure in patient with overdose), does not have other signs of hemochromatosis (bronze diabetes, cardiomyopathy, etc.), LFTs incompatible with primary biliary cirrhosis (would be cholestatic with direct hyperbilirubinemia and alk phos >> ALT/AST, and no signs of hepatitis on labs (negative HCV antibody and negative HB surface antigen)
  • Key idea: NASH has overtaken alcoholic steatohepatitis as most common cause of chronic liver disease and cirrhosis in certain countries

42. A 25-year-old man comes to the physician 6 hours after…

MRI of the thoracolumbar spine

  • Patient with type 1 diabetes (relative immunosuppression and increased risk of infection from use of needles for insulin administration) who presents with leg weakness and urinary retention in setting of fever, vertebral tenderness and leukocytosis, most consistent with epidural abscess pressing on the spinal cord
  • Key idea: In setting of low back pain, X-ray is indicated in setting of osteoporosis/compression fracture, ankylosing spondylitis or suspected malignancy
  • Key idea: In setting of low back pain, MRI is indicated in setting of sensory/motor deficits, cauda equina syndrome and suspected epidural abscess/infection

43. A 37-year-old woman comes to the emergency department…

Cardiac tamponade

  • Patient with history of breast cancer presents with progressive shortness of breath with elevated JVP, hypotension, pulsus paradoxus (drop in BP by at least 10 mm Hg on inspiration), enlarged heart on CXR and decreased voltage on ECG, most consistent with cardiac tamponade (likely secondary to cancer recurrence and metastasis to the pericardium)

44. A 25-year-old woman comes to the physician to discuss…

  • Key idea: 2 most important risk factors for Alzheimer’s is (1) Increased age (2) Family history of early-onset dementia (often due to mutation in Amyloid precursor protein or presinilin-1/2)
  • Key idea: Early-onset Alzheimer’s defined by symptoms appearing age 65 or younger

45. A 2-month-old boy is brought to the physician by his mother in…

Use of protective clothing

  • Key idea: Sunscreen not commonly used in infants under 6 months of age with best strategy to keep infants in the shade especially during the middle of the day and to use protective clothing

46. A 37-year-old man comes to the emergency department…

Echocardiography

  • Young man with recent URI presents with sharp substernal chest pain, fever and ECG with diffuse ST elevations and PR depressions most consistent with viral pericarditis which can be diagnosed with echocardiography because it is often complicated by pericardial effusion

Exam section 2:

1: A previously healthy 22-year-old man is brought to the emergency…

Exogenous ingestion of an organic acid

  • Young man presents with altered mental status found to have a severe anion-gap metabolic acidosis (142 – 102 – 12 = 28 > 12), with exogenous ingestion of an organic acid being the only answer that corresponds with this acid-base disturbance
  • Negative ketones in urine –> Rules out DKA
  • Excessive loss of bicarbonate = Type 2 renal tubular acidosis (non-anion gap metabolic acidosis)

2. A 37-year-old man is brought to the emergency department by…

Substance-induced psychotic disorder

  • Young previously healthy man with recent behavioral changes who presents with severe chest pain, signs of autonomic overactivity (tachycardia, hypertension, diaphoresis), rapid and pressured speech and ST-elevations on ECG, most consistent with substance-induced MI (most likely cocaine)
  • Key idea: In contrast to bipolar disorder (which can also lead to pressured speech, increased activity, etc.), substance-induced behavioral changes will also lead to pure physical manifestations of autonomic overactivity (hypertension, tachycardia, mydriasis, etc.)

3. A 32-year-old woman has had fatigue, fever and red bumps…

Penicillin G benzathine

  • Young woman with unsafe sexual practices and IVDU presents with fever, generalized lymphadenopathy and a diffuse maculopapular rash involving the palms and soles, most consistent with syphilis
  • Key idea: First-line treatment for syphilis is intramuscular penicillin G, with an alternative being doxycycline in patients with severe penicillin allergy
  • Key idea: Pregnant women with penicillin allergy should undergo penicillin desensitization because they are not allowed to use doxycycline (risk of teeth and bone abnormalities in baby)

4. A 37-year-old man comes to the physician 24 hours after the…

Small-bowel stricture

  • Young man with a chronic history of Crohn disease who presents with symptoms of a small bowel obstruction (distention, abdominal pain, high-pitched bowel sounds, nausea/vomiting, obstipation and air-fluid levels on small intestine), most concerning for a small-bowel stricture
  • Key idea: Crohn disease leads to transmural inflammation, and can therefore lead to strictures, fistulas, etc.

5. A previously healthy 17-year-old girl is brought to the emergency…

Spiral CT scan of the chest

  • Young patient on an OCP (leads to increased risk of clotting) with recent history of femur fracture and being bed-bound who presents with dyspnea, pleuritic chest pain, hypoxia and a new small pleural effusion most consistent with pulmonary embolism
  • Virchow’s triad for DVT/PE: (1) Hypercoagulability (2) Stasis (3) Endothelial injury

6. A 1-day-old male newborn had the onset of vomiting bilious…

Pancreatic enzyme

  • Newborn with family history of cystic fibrosis who has presentation consistent with meconium ileus (failure to pass meconium, bilious vomiting, inspissated tar-like meconium, etc.), and therefore should be treated with pancreatic enzyme
  • Key idea: Cystic fibrosis leads to fat malabsorption because pancreatic enzymes have trouble reaching the GI tract, so patients can benefit from pancreatic enzyme

7. A 32-year-old woman comes to the physician because of weakness…

Interferon beta therapy

  • Young woman presenting with neurological symptoms separated in time and space, including L’hermitte’s sign (electric-like sensation with neck flexion) and optic neuritis (unilateral painful vision changes/loss) with a brain MRI showing multifocal, ovoid subcortical white matter lesions in a periventricular distribution most consistent with multiple sclerosis
  • Key idea: This patient has not had symptoms for >24 hours and therefore can be transitioned from acute –> chronic treatment, with interferon beta being an option for chronic treatment
  • Key idea: Multiple sclerosis is treated chronically with immunosuppressants or interferon beta, whereas an acute MS flare should be treated with glucocorticoids or plasmapheresis

8. A 30-year-old primigravid woman with systemic lupus erythematous…

Complete atrioventricular block

  • Signs/symptoms of neonatal lupus include congenital heart block, rash and thrombocytopenia
  • Key idea: Only occurs if lupus patient has antibodies against SSA and SSB

9. A 52-year-old man who is HIV positive comes to the physician…

Squamous cell carcinoma

  • Middle-aged man with HIV presents with progressive pain and difficulty with bowel movements with a DRE showing a palpable mass in the anal canal with central ulceration most consistent with a squamous cell carcinoma
  • Key idea: HPV-related squamous cell carcinomas of the cervix, anus and penis are more commonly seen in HIV patients, which is why female patients with HIV should receive annual pap smears (rather than every 3-5 years)

10. A 19-year-old woman comes to the university health clinic because…

Blood: Trace / Protein: 1+ / RBC: 0-5 / WBC: >50 / Casts: None / Other: WBC clumps

  • Young sexually active woman (high risk of UTI) who presents with systemic signs of inflammation (fever) and is found to have dysuria, unilateral back pain and severe CVA tenderness, most consistent with pyelonephritis
  • Key idea: 2 major causes of WBC casts/clumps include pyelonephritis and Interstitial nephritis (analgesic nephropathy)

11. A 52-year-old woman with type 2 diabetes mellitus is brought to the…

Hyperosmolality

  • Patient with history of type 2 diabetes who has not been using medications presents with nausea/vomiting and confusion found to have a glucose level of 890 and a serum osmolality of 316, most consistent with Hyperosmolar hyperglycemic state
  • Key idea: Serum osmolality = (Na x 2) + (Glucose/18) + (BUN/2.6) + (Ethanol/4.6)
  • Key idea: The hyponatremia is secondary to the hyperosmolar state, and therefore is not the primary cause of confusion in this patient

12. A 20-year-old man is brought to the emergency department 30…

Retrograde urethrography

  • Young man with significant trauma found to have signs of pelvic fracture (pelvic pain with crepitus/tenderness over the pubis, x-ray findings) found to have a large scrotal hematoma, concerning for a urethral injury
  • Key idea: Although classically a pelvic fracture would injure the membranous portion of the urethra (leading to a high-riding prostate and blood at the urethral meatus), it can also damage the bulbar (spongy) urethra and lead to blood accumulation in the scrotum and blood at urethral meatus
  • Key idea: Retrograde urethrogram is the first test to perform in patients with suspected urethral injury

13. A 2-year-old African girl is brought to the physician because of…

Serum niacin concentration

  • Pellagra (niacin deficiency) commonly seen in patients with a corn-predominant diet and leads to Diarrhea, Dermatitis, Dementia and Glossitis

14. A 4-year-old girl is brought to the physician by her mother…

Medulloblastoma

  • 4 year old with an enlarged head circumference, signs of cerebellar dysfunction (ataxia, clumsiness, nystagmus, dysarthria) and upper motor neuron deficit of the bilateral lower extremities most concerning for medulloblastoma with drop metastases to the spinal cord
  • Key idea: Medulloblastoma arises in the cerebellar vermis and therefore leads to cerebellar dysfunction and can lead to “drop metastases” to the spinal cord through the CSF circulation

15. A 32-year-old woman comes to the physician after awakening…

Lumbar radiculopathy

  • Young otherwise healthy woman with severe dermatomal (L5) back pain after heavy lifting that increases with Valsalva and straight-leg raising, most consistent with a lumbar radiculopathy
  • Key idea: Two main causes of back pain with a positive straight leg test are disc herniation and osteophyte (same pathophysiology with spinal root compression)

16. A study is conducted to determine if a hemoglobin A1c…

Positive predictive value is 88%

  • Positive predictive value: Of the positive test results, how many actually had the disease (dependent on disease prevalence)
  • Sensitivity: Of the patients with disease, how many had a positive test result (independent of disease prevalence)

17. Two days after undergoing internal fixation of a femoral fracture…

Fat embolism

  • Key idea: Fat embolism often develops 24 to 72 hours after inciting event (long-bone fracture, orthopedic surgery, etc.)
  • Key idea: Fat embolism is a clinical diagnosis tht leads to triad of (1) Respiratory distress (2) Neurologic dysfunction (3) Petechial rash (or thrombocytopenia)

18. A 21-year-old woman comes to the physician because of a 3-day…

Epstein-Barr virus

  • Young patient with high-risk sexual behaviors presents with acute fever, sore throat, fatigue, tender/enlarged cervical lymph nodes, increased lymphocytes, and mildly increased LFTs with a negative rapid strep test, most consistent with infectious mononucleosis, which is caused by EBV > CMV

19. A previously healthy 18-year-old man is brought to the…

Diabetic ketoacidosis

  • Adolescent with history of weight loss despite normal appetite presents with acute nausea, vomiting, abdominal pain, mental status changes and is found to be dry on physican exam with labs consistent with an anion-gap metabolic acidosis (135 – 101 – 14 = 20 > 12) with an elevated glucose, most consistent with diabetic ketoacidosis
  • DKA: Glucose 250-500, anion-gap metabolic acidosis, ketones in urine, GI symptoms predominate
  • Hyperosmolar hyperglycemic state: Glucose > 600, relatively normal acid-base status, no ketones in urine, altered mental status predominates

20. A 17-year-old boy is brought to the emergency department…

Subclavian artery

  • Patient with a pulsatile hematoma = damage to an arterial structure
  • Based on location at the base of the neck, the most likely vessel involved is the subclavian artery
  • Note: If the thoracic aorta were damaged, the patient would likely have more severe hemodynamic collapse due to the amount of bleeding that would be produced

21. A 57-year-old man comes to the physician for a routine…

Fluid restriction

  • Patient with small cell lung cancer develops hyponatremia = SIADH until proven otherwise (basically confirmed with a urine osmolality > 100 and urine sodium > 40)
  • Mild hyponatremia symptoms (sodium>120, lethargy/forgetfulness): Fluid restriction +/- salt tablets
  • Severe hyponatremia symptoms (sodium<120, seizures, coma): Hypertonic (3%) saline

22. A previously healthy 62-year-old man comes to the physician…

Degenerative joint disease

  • Elderly man with joint pain in his knees and DIP joints of the hand that are worsened by use, most consistent with osteoarthritis
  • Key idea: The two main diseases that affect the DIP joints of the hand (at least on NBME exams) are osteoarthritis and psoriatic arthritis

23. A 72-year-old woman comes to the physician for a routine…

Cataracts

  • Elderly woman with trouble driving at night due to vision impairments related to bright lights who also has yellowing of the lenses on physical exam, most consistent with cataracts
  • Risk factors for cataracts: Advanced age, diabetes, corticosteroid use, smoking, excessive alcohol use, excessive sunlight exposure, congenital disease (galactosemia, etc.)

24. A 16-year-old boy is brought to the emergency department…

Peripheral muscarinic receptors

  • Organophosphates work by inhibiting acetylcholinesterase, thus leading to increased levels of acetylcholine that lead to overstimulation of parasympathetic functions and sweating (DUMBBELSS: Diarrhea, Urination, Miosis, Bradycardia, Bronchospasm, Emesis, Lacrimation, Sweating, Salivation) by binding to Muscarinic receptors and lead to neuromuscular overstimulation and eventual blockade due to binding to nicotinic receptors
https://courses.lumenlearning.com/epcc-austincc-ap1-2/chapter/divisions-of-the-autonomic-nervous-system/

25. A 9-year-old boy is brought to the physician by his mother…

FEV1: Decreased / FVC: Normal / TLC: Increased

  • Child with a family history of asthma who has presentation of cough-variant asthma (cough that is worse at night and with exercise), thereby leading to an obstructive pattern on PFTs (FEV1:FVC < 70% predicted, increased TLC)

26. A 20-year-old man comes to the student health services on…

Ceftriaxone and azithromycin therapy

  • Young man engaging in unprotected sex who presents with dysuria and scant urethral discharge
  • Key idea: If testing finds only chlamydia, then treat with only azithromycin
  • Key idea: If testing finds gonorrhea or you are treating empirically, then patient needs azithromycin + ceftriaxone

27. A 32-year-old primigravid woman at 27 weeks’ gestation is…

Cervical cancer

  • Pregnant woman with no prenatal care presents with painless vaginal bleeding and a friable ulcer on the cervix, concerning for cervical cancer
  • Key idea: For the NBME exam, if a patient has no prenatal care, then they want you to assume that the patient does not consistently engage in care and likely does not get regular pap smears
  • Note: Fundal placenta excludes placenta previa as a cause (important cause to consider in setting of painless 3rd trimester vaginal bleeding)

28. An 18-year-old man is brought to the emergency department…

Rocky Mountain spotted fever

  • Young man who has been camping for 3 weeks who presents with encephalopathy, fever/chills, headache and maculopapular rash involving the palms and soles along with petechiae/purpura of the lower extremities, concerning for RMSF
  • Key idea: RMSF (along with many other tick-borne illnesses such as Ehrlichiosis, Anaplasmosis, etc.) leads to leukopenia and thrombocytopenia
  • Key idea: Infectious rash over the palms and soles can be remembered with mnemonic CaRS: Coxsackie A, Rickettsia Rickettsii, Syphilis

30. An otherwise healthy 25-year-old nurse comes to the physician…

Hepatitis B immune globulin and hepatitis vaccine

  • Patient who has never received vaccination against hepatitis B is stuck with a needle by a patient with labs consistent with active hepatitis B infection, so patient should receive both hepatitis B vaccination series and hep B immune globulin
  • Infections that need to be treated with immunoglobulin: Dangerous infections To Be Healed Very Rapidly (Diphtheria, Tetanus, Botulism, Hepatitis B, Varicella, Rabies

31. A 47-year-old woman comes to the physician because of…

Chronic obstructive pulmonary disease

  • Middle aged patient with significant smoking history who is breathing with pursed lips (“pink puffer” of emphysema) and has diffuse wheezes in all lung fields, concerning for COPD
  • Aspiration of foreign body would only be acute, asthma would be episodic (not chronic progressive), bronchiectasis would be less likely to lead to diffuse lung changes and heart failure would lead to S4, elevated JVP, significant lower extremity edema, etc.

32. A 3-month-old boy is brought to the physician for evaluation…

Tetralogy of Fallot

  • Systolic ejection murmur in the left sternal border consistent with pulmonic stenosis (as seen in Tetralogy of Fallot) and history of becoming cyanotic while crying also consistent with Tetralogy of Fallot
  • Key idea: Causes of early cyanosis from primary heart defects can be remembered with mnemonic 1, 2, 3, 4 and 5 because it can be caused by (1) persistent truncus arteriosus (1 vessel) (2) transposition of the great vessels (2 switched vessels) (3) Tricuspid atresia (Tri = 3) (4) Tetralogy of Fallot (Tetra = 4) (5) Total anomalous pulmonary venous return (5 letters in name)
  • Key idea: Early cyanosis does NOT mean that patient needs to have cyanosis at birth

33. A biotechnology company sponsors a prospective study…

Selection bias

  • Patients should be randomly assigned to each type of graft in order to equally disperse potential confounding variables

34. An 11-month-old boy with Werdnig-Hoffmann spinal…

  • Patient with severe neuromuscular weakness who remains prone without much movement develops an ulcerative lesion on the back of their head, concerning for a decubitus ulcer
  • Note: Atypical mycobacterial infection can lead to ulcerative, violaceous cervical lymphadenopathy in children

35. A study is conducted to assess the accuracy of a new urine…

95%

  • Specificity = In patients who do NOT have the disease, what proportion will test NEGATIVE

36. A 52-year-old man is brought to the emergency department…

Perforation of the esophagus

  • Patient with severe retching/vomiting develops severe retrosternal pain and pain in the chest with a new pleural effusion on the left, most consistent with Boerhaave syndrome
  • Key idea: Severe retching can lead to Mallory-Weiss tear (severe hematemesis) and Boerhaave syndrome (severe chest pain with extrapulmonary air)

37. A previously healthy 47-year-old woman comes to the…

Plantar fasciitis

  • Key idea: History of heel pain that is worst with first steps in morning and pain with passive dorsiflexion of toes is classic for plantar fasciitis; plantar fasciitis also commonly associated with bone spur on plantar surface of foot
  • Differential for heel pain: (1) Plantar fasciitis (2) Bone infection or metastasis (constant pain that is worst at night) (3) Calcaneal stress fracture (worse with activity and palpation) (4) Tarsal tunnel syndrome (percussion tenderness and paresthesias over posterior tibial nerve in tarsal tunnel) (5) Achilles tendinopathy (pain/tenderness at posterior heel with palpable thickening of tendon that can be reproduced with foot dorsiflexion)

38. A 3-day-old newborn is brought to the emergency department…

Closure of the ductus arteriosus

  • This is a classic presentation for a patient with coarctation of the aorta. When they are first born, blood is able to get past the coarctation because blood flow through the ductus arteriosus enters the aorta distal to the coarctation. When the ductus arteriosus closes in the first week of life, the left side of the heart then sees a massive increase in afterload, leading to heart failure

39. A previously healthy 32-year-old woman comes to the…

Lymphangitis

  • Patient with an inflammed area of the distal radius with red streaks extending from the area towards the elbow, most consistent with lymphangitis
  • Key idea: Most common pathogens are MSSA and Group A strep, and therefore empiric treatment is cephalexin
https://en.wikipedia.org/wiki/Lymphangitis

40. A 16-year-old girl comes to the physician for a follow-up…

ACE inhibitor therapy

  • Adolescent with type 1 diabetes found to have microalbuminuria on labs, who should therefore receive ACEi/ARB to reduce risk of kidney damage
  • Key idea: ACE inhibitors or ARBs should be initiated at onset of microalbuminuria in all diabetic patients

41. Two hours after vaginal delivery at term complicated by…

Supplemental oxygen

  • Meconium aspiration is a common cause of persistent pulmonary hypertension of the newborn because it can lead to pulmonary vasoconstriction, which can lead to decreased pulmonary blood flow that leads to a vicious cycle leading to increased right to left shunting and increased PVR
  • To help combat this pulmonary vasoconstriction, the infant should be treated with supplemental oxygen (increased/improved oxygenation leads to dilation of lung capillaries and decreased pulmonary vascular resistance)

42. Eight hours after a total abdominal hysterectomy for…

Ureteral ligation

  • Young woman with recent total abdominal hysterectomy who is anuric with drain creatinine = serum creatinine, AKI and mild bilateral hydronephrosis consistent with ureteral ligation/damage due to surgery
  • Key idea: Drain fluid creatinine: Serum creatinine ratio > 1 consistent with urine leak
  • Key idea: Hysterectomy and other female GU operations are highly associated with ureteral damage (especially in NBME exams)

43. A 7-year-old girl is brought to the physician 3 days after her…

Topical mupirocin

  • Child presents with a rash on her mouth and cheek with honey-colored crust, most consistent with impetigo
  • Key idea: HSV infection can also present as a honey-color crusted rash on face, but it will be limited to the orolabial region (would not involve the cheek)

44. A 5-year-old boy is brought to the physician because of a 12-hour…

Factor VIII deficiency

  • Child with an maternal uncle with hemophilia who presents with joint swelling after minor trauma (concerning for hemarthrosis) with increased aPTT that corrects with mixing, concerning for hemophilia A (Factor VIII deficiency)
  • Key idea: Hemophilia A (X-linked, Factor 8 deficiency) > Hemophilia B (X-linked, Factor 9 deficiency) > Hemophilia C (Autosomal recessive, Factor 11 deficiency)
  • Key idea: If the coagulation studies are prolonged and do NOT correct with mixing, that should raise your suspicion for a coagulation factor inhibitor (often acquired in setting of recurrent transfusions, antiphospholipid syndrome, etc.)

45. A 2-hour-old female newborn has the onset of fever. She was born…

Intravenous administration of ampicillin and gentamicin

  • Newborn delivered by a mother with signs of chorioamnionitis (uterine tenderness and fever) has fever, tachypnea, and ill appearance, most consistent with neonatal sepsis
  • Key idea: All neonates with concern for sepsis should be empirically treated with ampicillin and gentamicin
  • Key idea: Mothers with postpartum endometritis are treated with ampicillin + gentamicin + clindamycin (only difference between the two)

46. A randomized, double-blind, controlled study is conducted to compare…

Variability among observers

  • Described study is subject to observer/measurement bias because each individual physician is making a subjective determination of the patient’s respiratory distress without objective measures such as O2 status, O2 need, respiratory rate, etc.

Exam section 3:

1: A 52-year-old woman comes to the physician because of a 2-month…

Decreased lactase activity in the brush border

  • Middle-aged woman experiencing increased abdominal cramps and diarrhea associated with increased milk consumption, consistent with lactose intolerance
  • Key idea: Pathophysiology is that many people lose activity of the lactase enzyme on the brush border of their small bowel as they become adults, such that lactose can no longer be broken down into glucose and galactose and absorbed (remember that small bowel can only accept carbohydrate monomers)
  • Key idea: Lactose hydrogen breath test would be positive because the lactose would not be absorbed in the small bowel and would therefore travel into the large intestine, where there are many bacteria that will break down the lactose via anaerobic glycolysis, leading to release of hydrogen gas (and cause of bloating in patients)

2. A 22-year-old primigravid woman delivers a 3200 g (7-lb 1-oz)…

21-Hydroxylase

  • XX newborn with ambiguous genitalia and elevated 17-hydroxyprogesterone levels, most consistent with 21-hydroxylase
  • Key idea: Three main enzymes that can be deficient in adrenal cortex pathway include (1) 21-hydroxylase (increased testosterone, decreased cortisol, decreased mineralocorticoids) (2) 11-hydroxylase (increased testosterone, decreased cortisol, normal/increased mineralocorticoids because 11-deoxycorticosterone has mineralocorticoid activity (3) 17-hydroxylase (decreased testosterone, decreased cortisol, increased mineralocorticoids)
https://www.merckmanuals.com/professional/pediatrics/endocrine-disorders-in-children/congenital-adrenal-hyperplasia-caused-by-21-hydroxylase-deficiency

3. A 25-year-old nurse comes to the physician 2 days after a PPD skin…

Isoniazid (INH) therapy only

  • Patient with a positive PPD (>10 mm in healthcare workers) who is asymptomatic and without CXR changes, most consistent with latent tuberculosis
  • Key idea: 3 treatment options for latent tuberculosis are (1) Isoniazid and rifapentine weekly for 3 months (2) Isoniazid for 6-9 months (3) Rifampin for 4 months
  • Key idea: PPD induration (NOT erythema) required for positivity depends upon patient risk factors with >5 mm being positive in patients with significant immunosuppression (HIV, organ transplant, immunosuppressant meds), recent contact with patient with active TB or patients with CXR findings consistent with TB // >10 mm being positive for patients from Tb endemic countries, IVDU, residents of high-risk settings (prisons, nursing homes, homeless shelters, etc.), children < 4 years old and patients working in mycobacterial labs // >15 mm in all patients

4. A 42-year-old woman comes to the physician because of a 1-year…

Serum antinuclear antibody assay

  • Middle-aged woman with dry mouth, dry eyes, parotitis and dental cavities, consistent with Sjogren syndrome (although classically associated with anti-SSA/SSB, patients are also commonly ANA positive)
  • Sjogren syndrome: Onset in middle age, positive ANA, and often systemic manifestations (malaise, weight loss, joint pains, etc.)
  • Age-related sicca syndrome: Onset in elderly patient, especially women with diabetes or thyroid disorders, no systemic symptoms

5. A 23-year-old woman comes to the physician because of a 3-day

Switch to a different method of contraception

  • Condoms/diaphragms are associated with increased risk of UTIs in young woman because they lead to increased friction
  • Key idea: Two other common methods of decreasing UTI risks are included in question stem (postcoital voiding, increased fluid intake)

6. A 67-year-old woman comes to the physician because of a 6-month…

Chronic inflammation

  • Elderly woman with rheumatoid arthritis presents with pallor and is found to have a microcytic anemia with a decreased TIBC (indicative of increased ferritin stores), most consistent with anemia of chronic disease
  • Iron deficiency (microcytic anemia): Iron labs show low iron, increased TIBC and decreased ferritin (often seen in young women due to menses and in older patient is colon cancer until proven otherwise)
  • Lead poisoning (microcytic anemia): Patient with exposure who has abdominal pain, neurologic symptoms, etc.
  • Key idea: While rheumatoid arthritis can be associated with anemia of chronic disease, osteoarthritis is NOT associated with anemia of chronic disease

7. A 23-year-old woman comes to the physician because of a 10-day…

Postviral myocarditis

  • Patient with history of influenza-like illness ~1 month ago who presents with acute dyspnea, fatigue, light-headedness, ECG changes and echo showing hypokinesis with normal valves, most consistent with a postviral myocarditis
  • Key idea: On NBME exams, if a young patient presents with signs of cardiogenic shock without trauma, a common cause is myocarditis
  • Acute constrictive pericarditis would be seen in patient with radiation to the chest, open heart surgery or some other insult that would lead to fibrosis of the pericardium

8. A 19-year-old man comes to the emergency department because he…

Dopamine

  • Patient with chronic history of hallucinations, delusions, agitation most consistent with schizophrenia
  • Key idea: Schizophrenia due to increased dopamine in the mesolimbic and mesocortical tracts, whereas Parkinson’s is due to decreased dopamine in the nigrostriatal pathway

9. A 37-year-old woman with type 1 diabetes mellitus comes to the…

Infection with Pseudomonas aeruginosa

  • Young woman with type 1 diabetes who has recent recreational water exposure who presents with a severe form of otitis externa (fever, ill-appearing), most consistent with necrotizing otitis externa
  • Key idea: Necrotizing otitis externa often seen in patients with diabetes mellitus and should be treated with intravenous antipseudomonal antibiotics and surgical debridement if antibiotics are unsuccessful
  • Key idea: In an NBME stem, if it says that there is purulent fluid in the ear canal and that the tympanic membrane cannot be visualized, that tells you that the insult is in the ear canal

10. A previously healthy 67-year-old woman comes to the physician…

Carotid duplex ultrasonography

  • Elderly woman who presents after an episode of amaurosis fugax (painless transient monocular vision loss caused by a small embolus in the ophthalmic artery) with hollenhorst plaques and an area of ischemia in the optic disc
  • Key idea: Amaurosis fugax is highly associated with carotid artery stenosis with resulting embolization, so patients should be worked up with a carotid artery ultrasound

11. A 12-year-old boy has the sudden onset of vomiting and moderate…

CT scan of the abdomen

  • Adolescent in a traumatic accident who presents with severe epigastric tenderness, nausea, vomiting, hypotension and tachycardia, normal abdominal ultrasound and normal LFTs, most consistent with duodenal hematoma > traumatic pancreatitis; in either case, the best next step would be CT scan of the abdomen
  • Key idea: CT scan is better than ultrasound in setting of pancreatitis, especially when etiology is not gallstones
  • Key idea: Morphine and other opioids (except meperidine) can lead to sphincter of Oddi dysfunction, which can further increase risk for pancreatitis

12. A 22-year-old primigravid woman at 10 weeks’ gestation is…

Lidocaine therapy

  • Key idea: Anesthetic that is accidentally injected into the blood stream can lead to systemic signs/symptoms (tingling, numbness, tinnitus, metallic taste, tachycardia, hypertension, etc.)

13. A 72-year-old man comes to the physician because of constipation…

Increased parathyroid hormone-related peptide concentration

  • Elderly man with known squamous cell carcinoma who presents with elevated calcium and otherwise normal labs, most consistent with PTHrP production
  • Key idea: Squamous cell carcinoma, breast cancer, renal cell carcinoma, ovarian cancer and bladder cancer are known for producing PTH-related peptide, which has the same effects as PTH (increased calcium, decreased phosphate)
  • Key idea: If answer was metastases to bone (commonly seen in breast cancer, multiple myeloma, prostate cancer) then patient would have back pain and/or imaging with osteolytic changes
  • Key idea: Lymphoma leads to hypercalcemia by leading to increased vitamin D levels

14. An 87-year-old woman is brought to the physician by her…

Clindamycin

  • Elderly woman with history of cerebral infarction who has aphasia and loss of gag reflex who presents with respiratory distress found to have RLL consolidation, most consistent with aspiration pneumonia
  • Key idea: Aspiration pneumonia caused by anaerobic organisms (because our mouth and GI tract has anaerobes), so patient is often treated with clindamycin which has good anaerobic coverage (other potential options are metronidazole or amoxicillin + clavulanate)

15. An 18-year-old woman has had difficulty swallowing solid food…

Achalasia

  • Young woman with difficulty swallowing and oropharyngeal dysphagia found to have classic “bird-beak” barium swallow finding of achalasia

16. A 37-year-old woman comes to the physician because of a…

Increased catecholamine production

  • Young otherwise healthy woman with episodic headaches, palpitations, and sweating found to have severely elevated blood pressure, most consistent with a pheochromocytoma (tumor of adrenal medulla leading to increased catecholamine production)
  • Key idea: Classically leads to episodes of the 5 P’s: elevated blood Pressure, Painful headache, Perspiration, Palpitations and Pallor

17. A 3-year-old girl is admitted to the hospital because of a 7-day…

Intravenous immune globulin

  • Key idea: Kawasaki disease characterized by fever of at least 5 days with at least 4/5 of the CRASH: Conjunctivitis, Rash, cervical Adenopathy, Strawberry tongue, Hand and foot edema
  • Key idea: Kawasaki disease treated with IVIG + high-dose aspirin (only time aspirin is given to children)

18. A 47-year-old man comes to the physician because of a 6-day…

Histoplasmosis

  • Key idea: Caves and Ohio are both buzzwords for Histoplasmosis
  • Key idea: Chest x-ray with bilateral lymphadenopathy and calcified nodules can be seen in tuberculosis, endemic mycoses (histo, blasto, cocci) and sarcoidosis

19. A 13-year-old boy is brought to the physician by his father…

Reassurance that the patient’s pubertal development is normal

  • Key idea: Patient has Tanner Stage 2 genitalia (testicular enlargement) and therefore does not meet criteria for pubertal delay and is within normal limits for pubertal development
  • Pubertal delay in boys: Lack of testicular enlargement (Tanner stage 2) by age 14
  • Pubertal delays in girls: Lack of breast development by 13 or lack of menarche by 15

20. A 32-year-old man comes to the physician because of a progressive…

Abnormal chloride channel in airway epithelium

  • 32 year old man with nasal polyps, chronic cough that is progressive with copious sputum production who has clubbing and obstructive PFT findings (FEV1:FVC ratio < 70%) found to have Pseudomonas aeruginosa colonization most consistent with bronchiectasis in the setting of cystic fibrosis
  • Key idea: Bronchiectasis and chronic bronchitis (type of COPD) are two conditions most known for copious sputum production
  • Potential etiologies of bronchiectasis: (1) Congenital: Cystic fibrosis, Kartagener’s (2) Chronic/prior infection: Aspergillosis, mycobacteria (3) Rheumatic disease: Rheumatoid arthritis, Sjogren’s (4) Immunodeficiency: Hypogammaglobulinemia (5) Airway obstruction: Cancer
  • Key idea: Bronchiectasis should be worked up with CF testing, PFTs, sputum culture, immunoglobulin levels and high-resolution CT of the chest

21. A 42-year-old man is brought to the physician by his wife because…

Caudate nucleus

  • Young patient with a father who developed a tic and committed suicide in middle-age who presents with progressive facial grimacing and signs of dementia, most consistent with Huntington disease (which is due to atrophy of the caudate nucleus)
  • Key idea: Huntington disease leads to triad of (1) Chorea (2) Dementia (3) Depression (and other psychosocial issues such as psychosis)
  • Atrophy of substantia nigra = Parkinson’s disease

22. A randomized trial is conducted to assess the effectiveness of a…

There is a 95% probability that the difference between the two cure rates is between 8% and 26%

  • Key idea: When we set alpha value to 5%, we are saying that there is a 5% chance of committing a type 1 error, which is where NO difference exists between the two treatments/groups/etc. but we do detect a difference (basically the chance that we detect a difference when one does not exist)

23. A 17-month-old infant with sickle cell disease has a 1-day history…

Ceftriaxone, parenterally

  • Infant with sickle cell disease presenting with sepsis (fever, clear rhinorrhea, lymphocytosis), which is most likely caused by Strep pneumo and can be effectively treated with ceftriaxone
  • Key idea: Sickle cell disease leads to functional asplenia, which leads to increased risk of encapsulated infections (SHiN: Strep pneumo, Haemophilus influenzae, Neisseria meningitidus)
  • Key idea: Although daily penicillin prophylaxis decreases the risk of Strep Pneumo infection, it does not completely eliminate the risk

24. An obese 14-year-old boy is brought to the physician because of…

Pin fixation

  • Obese adolescent who presents with atraumatic, afebrile groin pain and a limp with decreased range of motion and imaging showing displacement of the femoral head from the growth plate, most consistent with a slipped capital femoral epiphysis (SCFE) which is treated with pin fixation to avoid avascular necrosis
  • Key idea: Hip problems (osteoarthritis, avascular necrosis of the femoral head, etc.) often lead to groin symptoms
  • 2 common causes of avascular necrosis in child: (1) Legg-Calve Perthes disease: Commonly seen in males between 5-7 years old, X-ray often NORMAL, Idiopathic (2) SCFE: Commonly seen in obese ~12 year old child, Diagnosed via abnormal x-ray, Caused by epiphysis displacing relative to femoral neck and needs to be corrected surgically
https://www.mountnittany.org/articles/healthsheets/7381

25. A 77-year-old man is brought to the physician by his family…

Assessment for suicide risk

  • Elderly patient with significant smoking history who presents with cachexia (secondary to decreased appetite), decreased interest in smoking, abnormal mental status examination, slow speech, and cognitive dysfunction concerning for lung cancer vs. major depressive disorder, but patient has a normal chest x-ray which is more consistent with major depressive disorder and therefore patient should have suicide risk assessment (performed in all patients with depression)
  • Key idea: Major depression leads to at least 5 of the depressed mood + SIGECAPS symptoms for at least 2 weeks

26. A 57-year-old woman comes to the physician because of a…

Superficial thrombophlebitis

  • Middle-aged patient with smoking history who has developed tenderness, induration, erythema and thickening in a linear distribution on the extremities, most consistent with superficial thrombophlebitis in the setting of pancreatic cancer
  • Key idea: Pancreatic cancer patient will almost always be a smoker in NBME question stem and unique signs/symptoms associated with pancreatic cancer include (1) Back pain + weight loss (2) Migratory thrombophlebitis (Trousseau syndrome) (3) Obstructive jaundice with palpable, non-tender gallbladder (Courvoisier sign) (4) Diabetes in a patient without other risk factors (older patient who is losing weight but develops diabetes)
  • Lymphangitis would also lead to streaks, but would be less likely to lead to thickening/induration and more likely to lead to fever/chills
  • Varicose veins can be sensitive, but are not often tender or erythematous

27. A previously healthy 47-year-old woman comes to the physician…

Graves disease

  • On NBME exams, exophthalmos should always make you think of Graves disease (especially in the right demographic of a young or middle-aged woman)
  • Key idea: All causes of hyperthyroidism can lead to lid lag and lid retraction secondary to sympathetic activation of the superior tarsal muscle, but Graves specifically leads to exophthalmos and impaired extra-ocular motion due to glycosaminoglycan deposition behind the eyes

28. A 62-year-old man comes to the physician because of a…

Calculation of ankle brachial index

  • Elderly patient with history consistent with claudication (cramping pain in calf relieved with rest), which is best worked up with ankle-brachial index (better than arteriography for initial work-up because arteriography is much more invasive)
  • Key idea: ABI < 0.90 is abnormal
  • Claudication: Patient will have risk factors for peripheral vascular disease (diabetes, HTN, smoking, etc.), reduced lower extremity pulses, reduced lower extremity temperature, pain classically in the calves, reduced hair on legs
  • Pseudo-claudication (spinal stenosis): Positional (improves with flexion), classically affects buttocks and thighs, may be associated with back pain

29. A 54-year-old man comes to the physician because of impotence…

Increased iron stores in hepatocytes

  • Middle-aged man with signs of hypogonadism, diabetes (polydipsia, polyuria, weight loss, elevated glucose, glucosuria), cirrhosis without alcohol use history (spinder angiomata, increased bilirubin, decreased albumin, mildly elevated LFTs), and bronzing of the skin, most consistent with hemochromatosis
  • Mallory bodies is most consistent with alcoholic hepatitis

30. A 23-year-old woman comes to the physician because of an…

Diabetes mellitus

  • Diagnosis requires 2 out of 3 of the following criteria: (1) Clinical and/or biochemical hyperandrogenism (2) Oligomenorrhea (3) Polycystic ovaries on pelvic ultrasound /// Our patient has clinical hyperandrogenism (coarse hair over body) and oligomenorrhea (irregular menses)
  • Key idea: Although often associated with insulin resistance and ultimately associated with development of Type 2 diabetes, insulin resistance is not formally included in diagnostic criteria

31. An 82-year-old woman comes to the emergency department 2…

Heparin therapy

  • An elderly woman with atrial fibrillation who is not on anticoagulation presents with an acutely cold, painful foot with absent pulses, most consistent with critical limb ischemia secondary to an embolus from the left atrial appendage in setting of afib
  • Key idea: Patients with suspected acute limb ischemia should be treated with immediate initiation of anticoagulation, most often IV heparin

32. A 12-year-old boy is brought to the physician for a routine…

Restrict his intake of phosphorous

  • Normally the kidneys can handle phosphorous, but in setting of renal failure they cannot and it can lead to build-up of phosphorous that can lead to a variety of deleterious effects, such as metastatic calcification (which can increase risk of stroke, heart attack, etc.) and weak bones (because phosphorous pulls calcium out of bones); these complications are particularly important for kids because they progress chronically over time

33. A 37-year-old woman, gravida 4, para 4, has continued to have…

Genital tract laceration

  • Most common cause of post-delivery bleeding is uterine atony (especially in setting of prolonged, difficult delivery), but the uterine fundus would be described as soft and boggy rather than firm; therefore the next most common cause given that the placenta has been delivered would be trauma (especially with difficult delivery)
  • Causes of post-delivery bleeding is 4 T’s: (1) aTony (most common) (2) Trauma (3) Thrombin (coagulopathy) (4) Tissue (retained placenta, etc.)

34. A 67-year-old man comes to the physician 2 weeks after…

Excisional biopsy with narrow margin

  • Elderly patient with significant sun exposure (landscaper, farmer, etc.) with an ulcerated, shiny lesion on their chest most consistent with a squamous cell carcinoma or basal cell carcinoma
  • Key idea: Concerning skin lesions should be fully excised with ~3 mm margins unless the lesion is very large or in a location where complete excision may be problematic (ear, face, etc.)

35. A healthy 4-month-old boy is brought to the physician for a…

Family history

  • Key idea: Risk factors for asthma include (1) Family history of atopy (2) Environmental exposures (second-hand smoke, pollution, etc.)

36. A previously healthy 27-year-old man comes to the physician…

Release of preformed thyroid hormone

  • Young man with signs of hyperthyroidism (palpitations, heat intolerance, decreased TSH, increased T3/T4) who has a tender thyroid (consistent with de Quervain thyroiditis) and also has decreased iodine uptake into thyroid gland (consistent with thyroiditis, where preformed thyroid hormone is released but thyroid gland is not over-producing thyroid hormone)
  • The diagnostic algorithm outlined below is high-yield for NBME exams!
https://www.aafp.org/afp/2016/0301/p363.html

37. A 62-year-old man has dyspnea on exertion and a cough productive…

Polyvalent influenza vaccine, annually

  • Key idea: All patients should receive an annual flu vaccine
  • Patients should get a Tdap vaccine as an adult and then a Td booster every 10 years
  • All patients should get pneumococcal vaccines (PCV13 + PPSV23) at age 65, with patients under 65 with relative immunodeficiency or increased risk getting a PPSV23 (chronic heart, lung or liver disease; diabetes; current smokers; alcoholics) and patients under 65 with very high risk getting PCV13 + PPSV23 (CSF leak, sickle cell disease, asplenia, cochlear implants, HIV, chronic kidney disease, organ transplant, etc.)

38. A 42-year-old woman comes to the physician because of low-grade…

Biopsy and culture of the sinus mucosa for fungus

  • Patient with type 1 diabetes who presents with fever, headache and facial swelling with mild ketoacidosis and sinus pathology on CT scan, most concerning for Mucor infection
  • Key idea: Mucormycosis is most commonly seen in ketoacidotic diabetic patients and/or neutropenic patients
  • Key idea: Diagnosis requires biopsy and culture of the sinus mucosa because Aspergillus can less commonly lead to a similar presentation

39. A 22-year-old woman comes to the emergency department because…

Methotrexate therapy

  • Young woman with previously regular menses presents with a late menstrual period, RLQ abdominal pain with adnexal tenderness, and elevated beta-hCG with a complex adnexal mass on ultrasound, most consistent with uncomplicated ectopic pregnancy
  • Uncomplicated ectopic pregnancy (no hemodynamic instability or signs of rupture): Methotrexate
  • Complicated ectopic pregnancy (hemodynamic instability or signs of rupture): Urgent surgical exploration

40. A 10-year-old boy is brought for a routine examination. His…

Vitamin B12 (cyanocobalamin)

  • Child with resection of ileum (which is responsible for absorption of the ) who presents with macrocytic anemia, most consistent with vitamin B12 deficiency
  • Key idea: To remember the sections of small intestine and pertinent nutrient absorbed in each, use mnemonic Iron Fist Bro for Iron (duodenum), Folate (jejunum) and B12 (ileum)
  • Key idea: Ileum also responsible for absorption of bile salts, which are responsible for fat absorption and absorption of fat-soluble vitamins (A, D, E and K)

41. A 13-year-old boy is brought to the physician because a school…

Exposure to high-decibel sound

  • Patient who had normal hearing up until 3 years ago who has had potential barotrauma (snorkeling-related ear pain) and loud noises who has loss of high-frequency noises, most consistent with noise-related hearing loss
  • Key idea: Elderly patients who have had chronic exposure to noise will first lose high-frequency sounds (bilateral, symmetric sensorineural hearing loss), with young patients with increased exposure to loud noises being able to lose the high-frequency sounds due to destruction of hair cells at the cochlear base

42. A 57-year-old woman comes to the physician because of a 4-year…

Osteophytes

  • Presentation most consistent with osteoarthritis because the woman is heavy, the stiffness resolves quickly upon waking, the joint pain is asymmetric and, most importantly, it involves the DIP joints of the hand. The only two diseases that will involve the DIP joints are osteoarthritis and psoriatic arthritis. It is weird that it gets better as the day progresses, but the overall picture is most consistent with OA
  • Key idea: Osteoarthritis characterized by osteophytes, joint space narrowing, subchondral sclerosis and cysts

43. A 3-year-old girl is brought to the physician by her father because…

Uveitis

  • Child with atraumatic, progressive, bilateral knee pain with elevated ESR and otherwise normal exam/labs, most concerning for initial presentation of systemic juvenile idiopathic arthritis
  • Along with many other causes of autoimmune disease, SJIA is associated with anterior uveitis
  • Key idea: Classic NBME presentation is a patient with daily spiking fevers associated with a transient pink macular rash, along with signs of inflammation of labs (leukocytosis, anemia, thrombocytosis, increased ESR/CRP)

44. A 21-year-old college student comes to the physician for a…

“Tell me more about what you want to know”

  • Key idea: For the NBME exam communication questions, the most open-ended and patient-centered response is often correct

45. A 25-year-old man comes to the physician because of low back…

X-ray of the sacroiliac joints

  • Young man who presents with inflammatory back pain (worse in the morning but improves over course of the day) who has limited range of motion of the back (consistent with “bamboo spine” physiology), all of which is most consistent with ankylosing spondylitis that can be diagnosed with x-ray or MRI of the sacroiliac joints
  • Key idea: Indications for x-ray in setting of low back pain is (1) Osteoporosis or compression fracture (2) Suspected malignancy (3) Ankylosing spondylitis
  • Key idea: Indications for MRI in setting of low back pain is (1) Sensory/motor deficits (2) Cauda equina syndrome (3) Suspected epidural abscess or infection

46. A study is done to determine the effect of intracervical prostaglandin…

Low statistical power

  • Key idea: Increasing the sample size in a research study leads to a higher powered study
  • Other than sample size, the other aspects of the study are sound (randomization, exposure vs. control, etc.)

Exam section 4:

1: A 15-year-old boy is brought to the physician by his parents…

Bupropion

  • Patient with signs of major depressive disorder (too severe of a presentation for simple adjustment disorder), who should be treated with an anti-depressant, with Bupropion being the only antidepressant listed
  • First-line antidepressants: SSRIs, SNRIs, Bupropion, Mirtazapine
  • Benefits of bupropion: Mild stimulant effects, patient with comorbid cigarette use, helps patients with weight loss, favorable sexual side effect profile

2. A 67-year-old man comes to the physician because of 3-week…

Postrenal obstructive uropathy

  • Older patient with BPH with signs of an AKI (creatinine increase of at least 0.3) who also has suprapubic distention/tenderness, consistent with post-renal AKI in setting of urinary retention
  • Diabetic nephropathy can lead to chronic kidney disease over time, but it wouldn’t develop as quickly as seen in this patient

3. A 42-year-old woman comes to the physician because of increasing…

Resistant of kidneys to ADH (vasopressin)

  • Patient taking lithium (which is known to cause nephrogenic diabetes insipidus) who has developed polydipsia and polyuria who on water deprivation test is found to have very minimal increase in urine osmolality, most consistent with diabetes insipidus
  • Key idea: To formally differentiate between nephrogenic and central diabetes insipidus, you would give patient desmopressin (increased urine osmolality = central DI // No change in urine osmolality = nephrogenic DI), but in this case the history with lithium is suggestive of nephrogenic
  • Nephrogenic diabetes insipidus: Hereditary, hyperkalcemia, hypokalemia, lithium, demeclocycline
  • Central diabetes insipidus: Pituitary damage (tumor, autoimmune, trauma, surgery, ischemic encephalopathy)
https://biomedres.us/pdfs/BJSTR.MS.ID.000684.pdf

4. A 52-year-old man comes to the physician for a routine health…

MR angiography of the renal arteries

  • Middle-aged man with severe symptomatic hypertension with left abdominal bruit, most concerning for renal artery stenosis
  • Most common causes of renal artery stenosis include atherosclerosis, fibromuscular dysplasia and polyarteritis nodosa
  • Gold standard for renal artery stenosis is invasive angiography, whereas screening typically done with duplex ultrasound

5. A 32-year-old primigravid woman at 10 weeks’ gestation comes…

Abruptio placentae

  • Risk factors for placental abruption: Maternal hypertension, Maternal pre-eclampsia/eclampsia, prior placental abruption, cocaine use, tobacco use, abdominal trauma
  • Note: Maternal hypertension and pre-eclampsia also associated with increased risk of gestational diabetes
  • Maternal hypertension more likely to lead to oligohydramnios and small for gestational age baby (due to poor placental perfusion)
  • Risk factor for placenta previa includes previous placenta previa, previous c-section or uterine surgery

6. A 32-year-old woman, gravida 3, para 2, at 30 weeks’ gestation…

Placenta accreta

  • Pregnant woman with history of 2 C-sections currently in her 3rd trimester presents with painless vaginal bleeding found to have a placenta previa
  • Key idea: Placenta accreta occurs in up to 15% of cases of placenta previa, with both conditions having previous C-section as the major risk factor

7. A 57-year-old woman is brought to the emergency department 8…

Rectus sheath hematoma

  • Middle aged man recently started on anticoagulation who presents with progressive abdominal pain found to have a tender, mass next to the umbilicus, most consistent with a rectus sheath hematoma
  • Key idea: Often leads to anemia and leukocytosis
  • Key idea: Risk factors in addition to anticoagulation is abdominal trauma or forceful abdominal contractions (coughing, sneezing, etc.)

8. A 32-year-old man comes to the physician because of a 2-month…

Primary sclerosing cholangitis

  • Young man with ulcerative colitis who presents with fatigue, dark urine, mild jaundice, with a cholestatic pattern on LFTs (alk phos > ALT/AST), most consistent with primary sclerosing cholangitis
  • Primary sclerosing cholangitis: Classically seen in middle-aged men with ulcerative colitis, leads to beading of intra and extra-hepatic bile ducts on ERCP/MRCP, who has positive p-ANCA and elevated IgM
  • Primary biliary cholangitis: Classically seen in a middle-aged woman with other autoimmune diseases who has granulomas with lymphocytic infiltrate on biopsy, positive anti-mitochondrial antibody and elevated IgM

9. After an uncomplicated pregnancy, a 24-year-old primigravid…

Gonadal dysgenesis 45,X (Turner syndrome)

  • Baby born with excessive skin on back of neck, swelling of hands/feet and heart murmur which are classically seen in patients with Turner syndrome
  • Key idea: Noonan syndrome is basically the male version of Turner syndrome (although Noonan can also be seen in a female patient)

10. A 5-year-old girl is brought to the physician because of a 1-day…

Poststreptococcal glomerulonephritis

  • Young child who had strep throat 2 weeks ago who presents with periorbital edema, gross hematuria, proteinuria and RBC casts (consistent with glomerulonephritis), most consistent with post-strep glomerulonephritis
  • Both IgA and PSGN can lead to gross hematuria associated with an upper URI, with differences being (1) Acute PSGN: Seen 2-4 weeks after infection of pharynx or skin, Seen in children (often age 6-10), leads to low complement levels, patient will have subepithelial immune complex humps (2) IgA nephropathy: Seen concurrently with respiratory or GI infection, more common in young adult men (age 20-30), complement levels unaffected, patient will have IgA-based deposits in mesangium
  • Key idea: Treating strep throat will reduce risk of rheumatic fever, but does not reduce risk of post-strep glomerulonephritis

11. A healthy 37-year-old woman, gravida 4, para 4, comes to the…

Ectopic pregnancy

  • Patient with bilateral tubal ligation, which can increase risk of ectopic pregnancy (because if patient does inadvertently become pregnant, very high risk of ectopic pregnancy because tube is blocked)
  • Key idea: An estimated 1 in 200 patients will become pregnant after receiving tubal ligation, and patients who have had a tubal ligation and become pregnant are at increased risk of ectopic pregnancy compared to woman who have not had a tubal ligation and become pregnant

12. A healthy 52-year-old woman, gravida 3, para 3, comes to the…

Reassurance

  • Patient has classic presentation of a perimenopausal woman (menses become irregular, vaginal dryness, ~52 years of age)
  • Key idea: Perimenopausal woman can sometimes be treated with hormonal contraceptives to manage irregular bleeding, but this patient is over 35 and an active smoker, which is a contraindication to estrogen-containing OCPs
  • Key idea: Patient would be candidate for topical vaginal estrogen therapy (which does not carry same risks as systemic estrogen therapy)
  • Red flags requiring further work-up: Abnormal uterine bleeding (bleeding between periods, prolonged periods, or heavier bleeding during periods), especially in a patient with risk factors (obesity, PCOS, unopposed estrogen, etc.)

13. A 52-year-old woman comes to the physician because of…

Oral acyclovir

  • Middle-aged woman who presents with a vesicular, dermatomal rash with severe burning over the area, most consistent with herpes zoster
  • Key idea: Oral and IV acyclovir are equally effective in treating herpes zoster, so in this patient who does is going to be treated as an outpatient we would use oral acyclovir

14. A previously healthy 5-year-old boy is brought to the emergency…

Septic arthritis

  • Child with refusal to bear weight, fever, leukocytosis and increased ESR, consistent with septic arthritis
  • Septic arthritis: Leads to fever, ill-appearance, leukocytosis, failure to bear weight, synovial fluid WBC > 50,000 with positive gram stain and culture, treated with draining joint space + antibiotics
  • Transient synovitis: Patient will be afebrile, well-appearing between 2-6 years old, often had a previous viral URI, ability to bear weight, normal ESR, WBC count and synovial fluid analysis, condition is self-limited

15. A 52-year-old man is brought to the physician because of a 2-day…

Surgical debridement

  • Middle-aged man with diabetes who presents with severe pain in the scrotum, fever, and confusion with bullae and crepitus over the scrotum/perineum and a leukocytosis and elevated glucose concentration, most consistent with Fournier gangrene
  • Key idea: Fournier gangrene is a form of life-threatening necrotizing fasciitis of the perineum/scrotum most commonly seen in patients with obesity or diabetes mellitus who require urgent surgical intervention (should NOT be delayed for imaging)

16. A 57-year-old man is brought to the emergency department by…

Acute respiratory failure only

  • Middle-aged man with myasthenia gravis presenting with marked bilateral ptosis, progressive lethargy and decreased breath sounds (all of which point to a myasthenic crisis that would lead to respiratory insufficiency and CO2 retention), who has acid-base findings of acidemia with increased PCO2 and normal HCO3, consistent with an uncompensated respiratory acidosis
  • Key idea: If patient had chronic respiratory failure, HCO3 would be elevated because it would have had time to compensate over time

17. An 82-year-old woman is brought to the emergency department…

Acute myocardial infarction

  • Elderly patient with vasculopathic risk factors (hypertension, diabetes, smoking) presenting with acute shortness of breath, diaphoresis and pallor found to have hypotension and pulmonary edema with ECG findings of ST elevation in the anterolateral leads (I, avL, V2-V6), most consistent with myocardial infarction complicated by cardiogenic shock
  • Aortic dissection would lead to pain radiating to the back and X-ray would show widening of the mediastinum

18. A 62-year-old woman with hypertension comes to the physician…

Decreasing circulating angiotensin II concentration

  • Lisinopril is an ACE inhibitor that leads to decreased angiotensin 2, and therefore leads to decreased vasoconstriction, decreased Na/H20 reabsorption in the kidney, decreased aldosterone and decreased ADH, all of which can lead to decreased blood pressure

19. A 2977-g (6-lb 9-oz) female newborn is cyanotic 1 hour after…

Intravenous administration of prostaglandin

  • Newborn with cyanosis found to have a heart murmur and hypoxia unresponsive to supplemental oxygen, which is consistent with a severe intracardiac shunt)
  • Patient should be treated with alprostadil (or another prostaglandin analog) in order to keep the PDA open until the heart defect can be operatively managed

20. A 57-year-old man is brought to the emergency department 1…

Hypertension

  • Patient with Cushing’s triad (bradycardia, hypertension and irregular respirations) which is a sign of elevated intracranial pressure with a CT scan showing a high-density peri-ventricular hemorrhage, most consistent with a hypertensive bleed
  • Key idea: Common causes of brain bleeds include trauma, hypertension and cerebral amyloid angiography

21. A 27-year-old man comes to the physician because he has been…

Toxic megacolon

  • Young patient with concern for IBD (increased stool frequency, signs of inflammation [fatigue, weight loss], bloody stools) who presents with ill-appearance, fever, hypotension, abdominal distention and an X-ray showing dilated transverse colon, consistent with toxic megacolon in setting of ulcerative colitis
  • Key idea: Toxic megacolon associated with ulcerative colitis and C. diff colitis
  • Colonic pseudo-obstruction often seen in elderly patient who is post-op and/or received opiates, diverticulitis leads to LLQ pain in an elderly patient, and ischemic colitis leads to “pain out of proportion” in a patient with risk factors (atrial fibrillation, endocarditis, etc.)

22. A 15-year-old boy is brought to the physician by his mother 1…

Trichophyton rubrum

  • Teenage wrestler (wrestler should always make you think of tinea infection!) who has an erythematous, scaly, pruritic, annular rash with raised-edges most consistent with tinea corporis
  • The most common dermatophyte organisms include Trichophyton, Microsporum, and Epidermophyton

23. A 38-year-old woman has had severe nausea, vomiting, and…

Staphylococcus aureus

  • Causes of rapid-onset food poison (< 6 hours) are due to ingestion of pre-formed toxin and are most commonly caused by Staph aureus (cream-based food such as egg salad) or Bacillus cereus (reheated rice syndrome)
  • Clostridium perfringens (improperly cooked/stored meat) is another cause of food poisoning, but often occurs 6-24 hours after ingestion because patients ingest spores which need germinate in digestive tract to secrete toxin

24. A 62-year-old man comes to the physician to discuss the results…

The aneurysm should not be repaired because of the patient’s poor prognosis

  • Elderly man with imaging showing metastatic pancreatic cancer also found to have a stable abdominal aortic aneurysm
  • Median survival of metastatic pancreatic cancer is 3-6 months, so the patient would most likely die from cancer before having any negative effects from the aneurysm, so repair of aneurysm has risks > benefits for this specific patient
  • Indications for abdominal aorta aneurysm repair in healthy patient: (1) > 5.5 cm in diameter (2) expansion of at least 0.5 cm in 6 months and/or 1 cm per year (3) symptomatic (abdominal pain, flank pain, limb ischemia)

25. A 25-year-old woman comes to the physician for a follow-up…

Chlamydia trachomatis infection

  • Young woman with suspected UTI who has symptoms that are refractory to 2 courses of antibiotics and urethral tenderness on physicial exam, most consistent with urethritis which is most commonly caused by N. Gonorrhea or Chlamydia trachomatis
  • Key idea: On the NBME, when it says that a woman is sexually active and uses oral contraception, that means that they do not need condoms and are at increased risk for STIs
  • Key idea: Empiric urethritis treated with azithromycin + ceftriaxone, gonorrhea positive only treated with azithromycin + ceftriaxone and chlamydia positive only treated with azithromycin only

26. A 37-year-old woman comes to the physician because of a 3-month…

Renovascular hypertension

  • Young woman with hypertension, headaches and a renal arteriography showing an 80% obstructive lesion, most consistent with fibromuscular dysplasia leading to renal artery stenosis
  • Key idea: Although fibromuscular dysplasia most commonly leads to resistant hypertension due to renal artery involvement, cerebrovascular fibromuscular dysplasia also a common presentation (headache, tinnitus, dizziness, TIA, amaurosis fugax, etc.)

27. A 27-year-old woman at 6 weeks’ gestation comes to the physician…

Switch to insulin

  • Pregnant patient with type 2 diabetes with signs of poorly controlled blood glucose while on metformin, therefore requiring step-up in therapy to include insulin
  • Target blood glucose levels in gestational diabetes mellitus are fasting glucose < 95, 1-hour postprandial glucose < 140 and 2-hour postprandial glucose < 120
  • Key idea: Treatment options in gestational diabetes are (1st-line) Dietary modification (2nd-line) Insulin, metformin

28. A 42-year-old woman, gravida 5, para 4, at 14 weeks’ gestation…

Blighted ovum

  • Blighted ovum = Anembryonic pregnancy = fertilized egg attaches to uterine wall but the embryo does not develop during the first trimester

29. A 66-year-old woman has had lower abdominal pain and fullness…

Diverticulitis

  • Key idea: Diverticulitis classically leads to dull LLQ pain, nausea/vomiting, alteration in bowel habits and bladder symptoms (sterile pyuria, dysuria, etc.) +/- tender LLQ mass
  • Key idea: Diverticulitis treated with bowel rest and antibiotics (ciprofloxacin + metronidazole) and followed up 4-8 weeks later with colonoscopy (colonic malignancy can mimic the presentation and CT findings seen in diverticulitis)
  • Key idea: Indications for surgery in setting of acute diverticulitis (1) Emergency operation (peritonitis, abscess, etc.) (2) Any patient who has survived 2 episodes of acute diverticulitis should have elective removal of affected area to prevent recurrence

30. A 72-year-old man comes to the physician because of 2-year…

Spinal stenosis

  • Pain in the hips/thighs that is exacerbated by walking and improves with leaning forward with normal posterior tibial pulses, most consistent with pseudoclaudication due to spinal stenosis
  • Claudication: Patient will have risk factors for peripheral vascular disease (diabetes, HTN, smoking, etc.), reduced lower extremity pulses, reduced lower extremity temperature, pain classically in the calves, reduced hair on legs
  • Pseudo-claudication (spinal stenosis): Positional (improves with flexion), classically affects buttocks and thighs, may be associated with back pain

31. An 82-year-old nursing home resident is admitted to the hospital…

Peripheral vasodilation

  • Elderly nursing home patient presents with altered mental status, fever, hypotension, warm extremities and a positive urinalysis (pyuria if >10 WBC/hpf), most consistent with urosepsis (infection leading to peripheral vasodilation and distributive shock)
  • Key idea: Distributive shock (anaphylaxis, sepsis, CNS injury) is the only type that will lead to warm and dry skin
  • Key idea: Patient needs to have symptoms in order to be diagnosed with UTI > Asymptomatic bacteriuria in the setting of a positive urinalysis, but in addition to dysuria and suprapubic tenderness, altered mental status is often attributed to UTI and considered a symptom

32. A 52-year-old woman comes to the physician as a new patient

Decreased intake of alcohol

  • Modifiable risk factors for osteoporosis: Excessive alcohol intake, sedentary lifestyle, smoking
  • Non-modifiable risk factors for osteoporosis: Advanced age, postmenopausal woman, low body weight

33. A 20-year-old man with a 2-year history of hepatolenticular…

Zinc

  • Signs of zinc deficiency: Alopecia, dermatitis, scaly skin around mouth/eyes, abnormal taste, impaired wound healing
  • Signs of copper deficiency: Hematologic abnormalities (microcytic anemia, leukopenia) and myeloneuropathy
  • Iron deficiency: Microcytic anemia

34. A full-term 8-hour-old male newborn regurgitates his first feeding…

Insertion of a radiopaque nasogastric tube

  • Newborn who has trouble swallowing and handling secretions, most consistent with a suspected tracheoesophageal fistula which can be tested by passing an NG tube

35. A 19-year-old woman, gravida 2, para 1, comes for a follow-up…

Ultrasonography to confirm gestational age

  • Key idea: Most common causes of elevated AFP in a pregnant patient are under-estimation of gestational age, fetal demise, multiple gestations, ventral wall defects, neural tube defects and liver disease
  • Key idea: Next step in management of elevated AFP is ultrasound to assess gestational age, viability, number of pregnancies, and assess for defects

36. A previously healthy 2-year-old boy is brought to the emergency…

Drain cleaner

  • Key idea: Ingestion of household bleach is often considered a benign ingestion, whereas ingestion of industrial-grade bleach is very severe/morbid

37. A 2-week-old newborn is brought to the physician because of…

Normal thymus

  • Thymus is normal in children < 3 years of age and commonly leads to “sail sign” due to triangular shape
https://www.slideshare.net/airwave12/chest-x-ray-positioning

38. A 22-year-old woman has had heavy menstrual bleeding and easy…

Presence of anti-platelet IgG antibodies

  • Previously healthy young woman who presents with heavy menstrual bleeding and easy bruisability found to have thrombocytopenia with bone marrow aspirate showing increased number of megakaryocytes, with platelet count responsive to prednisone, most consistent with idiopathic thrombocytopenia
  • Key idea: Immune thrombocytopenia caused by platelet antibodies and is often associated with a preceding viral infection
  • Impaired platelet production = Aplastic anemia = Pancytopenia

39. A 47-year-old man comes to the physician for a routine health…

Schedule a follow-up examination in 1 month

  • Middle-aged man found to have elevated blood pressure for the first time, and therefore should be initially managed with a trial of lifestyle modifications

40. A previously healthy 21-year-old man is brought to the…

Psychiatric assessment

  • A previously healthy young man with recent life stressors who presents with sudden onset of blindness in one eye with a completely normal physical exam (including tests demonstrating that he can in fact see out of that eye), most consistent with conversion disorder
  • Testing will often involve a neurologist and psychiatrist

41. A 42-year-old woman comes to the physician because of a…

Fine-needle aspiration biopsy

  • Young woman with a nontender thyroid mass/nodule that is 2 cm in size, requiring fine needle aspiration biopsy
  • Key idea: All thyroid nodules should be worked up with a TSH and thyroid ultrasound, with thyroid nodules greater than 2 cm in diameter or thyroid nodules greater than 1 cm with high-risk sonographic features (irregular markings, microcalcifications, etc.) should undergo fine needle aspiration biopsy
  • Key idea: The thyroid gland is highly vascular, which is why you would always use a fine-needle aspiration biopsy for evaluation
  • Key idea: Mass in the neck that moves with swallowing = Thyroid or thyroglossal duct cyst
  • Key idea: Hypofunctional or “cold” thyroid nodule has a higher risk of malignancy as compared to a hyperfunctional “hot” thyroid nodule

42. A 52-year-old man comes to the physician because of a…

Polysomnography

  • Obese middle-aged patient with daytime fatigue and a likely reactive polycythemia (chronic hypoxia –> increased EPO production –> increased Hgb/Hct) in the setting of obstructive sleep apnea +/- obesity hypoventilation syndrome
  • Obstructive sleep apnea requires polysomnography for formal evaluation

43. A 16-year-old boy comes to the physician because he would…

“Circumcision cannot be done without the consent of your parents”

  • Medical emancipation: “Sex, drugs and rock and roll” = STIs, pregnancy care, or contraception // Mental healthy and addiction services // Emergency care
  • Legal emancipation: Parent, married, in the military, high school graduate, financially independent (i.e. lives on their own)

44. A 67-year-old woman comes to the physician 6 hours after a…

Carotid duplex ultrasonography

  • Elderly patient who presents with short episodes of left-sided weakness and right-sided blindness, consistent with embolization from an unstable plaque in the RIGHT carotid artery
  • Key idea: Amaurosis fugax = Episodic, temporary loss of vision often in one eye = carotid artery atherosclerosis (at least on NBME exams) = carotid duplex ultrasonography

45. A previously healthy 22-year-old woman comes to the physician…

Inflammation within the first dorsal extensor tendon compartment

  • Classic presentation of de Quervain tendinopathy, which is atraumatic lateral wrist pain in a woman 30-50, often seen 4-6 weeks postpartum due to repetitive thumb abduction and extension when lifting their infant
  • Key idea: Also commonly leads to increased pain with adduction (ulnar deviation) of the wrist due to stretching of the tendons over the radial styloid

46. A previously healthy 4-year-old boy is brought to the physician…

Upper respiratory tract infection

  • Unable to access audio, but patient has classic symptoms of a URI (fever, cough, rhinorrhea) and likely normal heart sounds (or a soft systolic ejection murmur)

We are not affiliated with the NBME, USMLE or AAMC.

The answer explanations may not be reproduced or distributed, in whole or in part, without written permission of Step Prep.